Family Med Board Questions - I
For normal term infants, current practice is to introduce solid foods into the diet at what age?
*4-6 months* the extrusion reflex (pushing foreign material out of the mouth with the tongue) makes feeding of solids difficult and often forced. This reflex disappears around the age of 4 months, making feeding easier. The introduction of solids at this age helps supply calories, iron, and vitamins, and may prepare the infant for later dietary diversity and healthy dietary habits.
A 3-year-old male presents with a 3-day history of fever and refusal to eat. Today his parents noted some sores just inside his lips. No one else in the family is ill, and he has no significant past medical history. He is up-to-date on his immunizations and has no known allergies. On examination, positive findings include a temperature of 38.9°C (102.0°F) rectally, irritability, and ulcers on the oral buccal mucosa, soft palate, tongue, and lips. He also has cervical lymphadenopathy. The remainder of the physical examination is normal. The child is alert and has no skin lesions or meningeal signs. Which one of the following would be the most appropriate treatment?
*Acyclovir (Zovirax) suspension* for *gingivostomatitis* due to a primary or initial infection with *HSV-1*). After a primary HSV-1 infection with oral involvement, the virus invades the neurons and replicates in the trigeminal sensory ganglion, leading to recurrent herpes labialis and erythema multiforme Antibiotics are *not* useful for the treatment of herpetic gingivostomatitis and could confuse the clinical picture should this child develop erythema multiforme, which occurs with HSV-1 infections. An orally applied corticosteroid is not specific treatment, but some might try it for symptomatic relief. An immunosuppressant is sometimes used for the treatment of Behçet's syndrome, but this patient's findings are not consistent with that diagnosis. Therefore, the only specific treatment listed is acyclovir suspension, which has been shown to lead to earlier resolution of fever, oral lesions, and difficulties with eating and drinking. It also reduces viral shedding from 5 days to 1 day
Which one of the following is most commonly implicated in interstitial nephritis?
*Antibiotics*, especially penicillins, cephalosporins, and sulfonamides, are the most common drug-related cause of acute interstitial nephritis. Corticosteroids may be useful for treating this condition. Other drugs, such as NSAIDs or ACE inhibitors, may cause renal injury, but not acute interstitial nephritis.
While evaluating a stroke patient, you ask him to stick out his tongue. At first he is unable to do this, but a few moments later he performs this movement spontaneously. This defect is known as
*Apraxia* is a transmission disturbance on the output side, which interferes with skilled movements. Even though the patient understands the request, he is unable to perform the task when asked, but may then perform it after a time delay. *Agnosia* is the inability to recognize previously familiar sensory input, and is a modality-bound deficit. For example, it results in a loss of ability to recognize objects. *Astereognosis* is the ability to recognize objects by palpation in one hand but not the other.
Which one of the following has been shown to be most effective for smokeless tobacco cessation?
*Behavioral interventions*, especially those including telephone counseling and/or a dental examination, have been shown to be helpful for promoting smokeless tobacco cessation (SOR B). Studies examining mint snuff as a tobacco substitute, bupropion, and nicotine replacement in patch or gum form did not show any significant benefit.
Metformin (Glucophage) should be stopped prior to which one of the following, and withheld until 48 hours after completion of the test?
*CT angiography*; since even a temporary reduction in renal function, such as occurs after pyelography or angiography, can cause *lactic acidosis* in patients taking metformin, the drug should be discontinued 48 hours before such procedures (SOR C) and restarted 48 hours after the procedure if renal function is normal.
A 65-year-old female who is morbidly obese presents to your office with intertrigo in the axilla. On examination you detect small, reddish-brown macules that are coalescing into larger patches with sharp borders. You suspect cutaneous erythrasma complicating the intertrigo. What would be the most appropriate topical treatment for this condition?
*Intertrigo* is inflammation of skinfolds caused by skin-on-skin friction and is common on opposing cutaneous or mucocutaneous surfaces. Secondary cutaneous bacterial and fungal infections are common complications. Cutaneous erythrasma may complicate intertrigo of interweb areas, intergluteal and crural folds, axillae, or inframammary regions. *Erythrasma* is caused by Corynebacterium minutissimum and presents as small reddish-brown macules that may coalesce into larger patches with sharp borders. Intertrigo complicated by erythrasma is treated with *topical or oral erythromycin*.
A 30-year-old previously healthy male comes to your office with a 1-year history of frequent abdominal pain, nonbloody diarrhea, and a 20-lb weight loss. He has no history of travel outside the United States, antibiotic use, or consumption of well water. His review of systems is notable for a chronic, intensely pruritic rash that is vesicular in nature. His review of systems is otherwise negative and he is on no medications. The most likely cause of his symptoms is?
*Celiac sprue*, an autoimmune disorder characterized by inflammation of the small bowel wall, blunting of the villi, and resultant malabsorption. *Symptoms*= diarrhea, fatigue, weight loss, abdominal pain, and borborygmus; *treatment* = elimination of gluten proteins from the diet. *Extraintestinal manifestations* are less common but may include elevated transaminases, osteopenia, and iron deficiency anemia. Serum IgA tissue transglutaminase (TTG) antibodies are highly sensitive and specific for celiac sprue, and a small bowel biopsy showing villous atrophy is the gold standard for diagnosis. This patient's rash is *dermatitis herpetiformis*, pathognomonic for celiac sprue and responds well to a strict gluten-free diet. Lactose intolerance, irritable bowel syndrome, collagenous colitis, and Crohn's disease are in the *differential diagnosis* for celiac sprue. However, significant weight loss is not characteristic of irritable bowel syndrome or lactose intolerance. The diarrhea associated with Crohn's disease is typically bloody. Collagenous colitis does cause symptoms similar to those experienced by this patient, but it is not associated with dermatitis herpetiformis.
Which is the greatest risk factor for abdominal aortic aneurysm (AAA)?
*Cigarette* smokers are five times more likely than nonsmokers to develop an abdominal aortic aneurysm (AAA). The risk is associated with the number of years the patient has smoked, and declines with cessation. *Diabetes mellitus* is protective, decreasing the risk of AAA by half. *Women* tend to develop AAA in their sixties, 10 years later than men. *Whites* are at greater risk than African-Americans. *Hypertension* is less of a risk factor than cigarette smoking (SOR A).
What antibiotic is contraindicated in the second and third trimesters of pregnancy?
*Doxycycline* is contraindicated in the second and third trimesters of pregnancy due to the risk of permanent discoloration of tooth enamel in the fetus. Cephalosporins such as ceftriaxone are usually considered safe to use during pregnancy. The use of ciprofloxacin during pregnancy does not appear to increase the risk of major congenital malformation, nor does the use of amoxicillin. Animal studies using rats and mice treated with daily doses of azithromycin up to maternally toxic levels revealed no impairment of fertility or harm to the fetus.
A 17-year-old white female has a history of anorexia nervosa, and weight loss has recently been a problem. The patient is an academically successful high-school student who lives with her parents and a younger sibling. Her BMI is 17.4 kg/m2 . Her serum electrolyte levels and an EKG are normal. Which one of the following interventions is most likely to be successful?
*Family-based treatment* for the adolescent with anorexia nervosa has been found to provide superior results when compared with individual adolescent-focused therapy (SOR B). Antidepressants have not been successful. They may be indicated for coexisting conditions, but this is more common with bulimia.
A 53-year-old male presents for a routine well-care visit. He has no health complaints. His wife has accompanied him, however, and is quite concerned about changes she has noticed over the last 1-2 years. She says that he has become quite apathetic and seems to have lost interest in his job and his hobbies. He has been accused of making sexually harassing comments and inappropriate touching at work, and he no longer helps with household chores at home. He often has difficulty expressing himself and his speech can lack meaning. The physical examination is normal. Based on the history provided by the wife, you should suspect a diagnosis of :
*Frontotemporal dementia (FTD)*, a common cause of dementia in patients younger than 65, with an insidious onset. Unlike with Alzheimer's disease, *memory is often relatively preserved*, even though insight is commonly impaired. There are three subtypes of frontotemporal dementia: behavioral variant FTD, semantic dementia, and progressive nonfluent aphasia. This patient would be diagnosed with the behavioral variant due to his loss of executive functioning leading to personality change (apathy) and inappropriate behavior (SOR C). Speech output is often distorted in frontotemporal dementia, although the particular changes differ between the three variants. Patients with FTD often are mistakenly thought to have major depressive disorder due to their apathy and diminished interest in activities. However, patients with depression do not usually exhibit inappropriate behavior and lack of restraint. Dementia with Lewy bodies and Alzheimer's dementia are both characterized predominantly by memory loss. Alzheimer's dementia is most common after age 65, whereas FTD occurs most often at a younger age. Lewy body dementia is associated with parkinsonian motor features. Patients diagnosed with schizophrenia exhibit apathy and personality changes such as those seen in FTD. However, the age of onset is much earlier, usually in the teens and twenties in men and the twenties and thirties in women.
Which one of the following has been shown to be effective for improving symptoms of varicose veins?
*Horse chestnut seed extract* has been shown to have some effect when used orally for symptomatic treatment of chronic venous insufficiency, such as varicose veins. It may also be useful for relieving pain, tiredness, tension, and swelling in the legs. It contains a number of anti-inflammatory substances, including *escin*, which reduces edema and lowers fluid exudation by decreasing vascular permeability. *Milk thistle* may be effective for hepatic cirrhosis. *Ephedra* is considered unsafe, as it can cause severe life-threatening or disabling adverse effects in some people. *St. John's wort* may be effective for treating mild to moderate depression.
An 88-year-old male has been hospitalized for the past 3 days after being found on the floor of his home by a neighbor and transported to the hospital by ambulance. He was cachectic and dehydrated at the time of admission, with a serum albumin level of 1.9 g/dL (N 3.5-4.7). He has received intravenous fluids and is now euvolemic. He began nasogastric tube feeding 2 days ago and has now developed nausea, vomiting, hypotension and delirium. Which one of the following is the most classic electrolyte abnormality with this condition?
*Hypophosphatemia* secondary to *Refeeding syndrome*, defined by the potentially fatal shifts in fluids and electrolytes that may occur in malnourished patients receiving artificial refeeding (whether enterally or parenterally). These shifts result from hormonal and metabolic changes and may cause serious clinical complications. The syndrome is complex and may also include abnormal sodium and fluid balance; changes in glucose, protein, and fat metabolism; thiamine deficiency; hypokalemia; and hypomagnesemia.
You see a 22-year-old female who sustained a right knee injury in a recent college soccer game.She is a defender and executed a sudden cutting maneuver. With her *right foot planted* and her ankle locked, she attempted to shift the position of her body to stop an oncoming ball and felt her *knee pop*. She has had a moderate amount of pain and swelling, which began within 2 hours of the injury, but she is most concerned about the *loss of knee hyperextension*. Which one of the following tests is most likely to be abnormal in this patient?
*Lachman sign* (soft end feel to the translation of the tibia as its pulled anteriorly) for ACL tears, which occur more commonly in women than in men. The intensity of play is also a factor, with a much greater risk of ACL injuries occurring during games than during practices. The most accurate maneuver for detecting an ACL tear is the Lachman test (sensitivity 60%-100%, mean 84%), followed by the anterior drawer test (sensitivity 9%-93%, mean 62%) and the pivot shift test (sensitivity 27%-95%, mean 62%) (SOR C). McMurray's test is used to detect meniscal tears.
A 38-year-old day-care worker consults you for "a cold that won't go away." It began with a runny nose, malaise, and a slight temperature elevation up to 100°F (37.8°C). She notes that after 2 weeks she is now experiencing "coughing fits," which are sometimes so severe that she vomits. She has had no immunizations since her freshman year in college and does not smoke. On examination you note excessive lacrimation and conjunctival injection. Her lungs are clear. Which one of the following is the most likely diagnosis?
*Pertussis*, once a common disease in infants, declined to around 1000 cases in 1976 as a result of widespread vaccination. The incidence began to rise again in the 1980s, possibly because the immunity from vaccination rarely lasts more than 12 years. The disease is characterized by a prodromal phase that lasts 1-2 weeks and is indistinguishable from a viral upper respiratory infection. It progresses to a more severe cough after the second week. The cough is paroxysmal and may be severe enough to cause vomiting or fracture ribs. Patients are rarely febrile, but may have increased lacrimation and conjunctival injection. The incubation period is long compared to a viral infection, usually 7-10 days. Nonasthmatic eosinophilic bronchitis, cough-variant asthma, and gastroesophageal reflux disease cause a severe cough not associated with a catarrhal phase. A rhinovirus infection would probably be resolving within 2-3 weeks.
Which one of the following treatments for diabetes mellitus reduces insulin resistance?
*Pioglitazone* (Actos) is a thiazolidinedione, which reduces insulin resistance. It is believed that the mechanism for this is activation of PPAR-Y, a receptor that affects several insulin-responsive genes. *Repaglinide* and nateglinide are nonsulfonylureas that act on a portion of the sulfonylurea receptor to stimulate insulin secretion. *Acarbose* is a competitive inhibitor of α-glucosidases, enzymes that break down complex carbohydrates into monosaccharides. This delays the absorption of carbohydrates such as starch, sucrose, and maltose, but does not affect the absorption of glucose. *Sitagliptin* is a DPP-IV inhibitor, and this class of drugs inhibits the enzyme responsible for the breakdown of the incretins GLP-1 and GIP. *Exenatide* is an incretin mimetic that stimulates insulin secretion in a glucose-dependent fashion, slows gastric emptying, and may promote satiety.
A 22-year-old white female comes to your office complaining of dizziness. She was in her usual good health until about 2 weeks before this visit, when she developed a case of gastroenteritis that other members of her family have also had. Since that time she has been lightheaded when standing, feels her heart race, and gets headaches or blurred vision if she does not sit or lie down. She has not passed out but has been unable to work due to these symptoms. She is otherwise healthy and takes no regular medications. A physical examination is normal except for her heart rate, which rises from 72 beats/min when she is lying or sitting to 112 beats/min when she stands. Her blood pressure remains unchanged with changes of position. Routine laboratory tests and an EKG are normal. What is the most likely cause of this patient's condition?
*Postural orthostatic tachycardia syndrome* (POTS) is manifested by a rise in heart rate >30 beats/min or by a heart rate >120 beats/min within 10 minutes of being in the upright position. Symptoms usually include position-dependent headaches, abdominal pain, lightheadedness, palpitations, sweating, and nausea. Most patients will not actually pass out, but some will if they are unable to lie down quickly enough. This condition is most prevalent in white females between the ages of 15 and 50 years old. Often these patients are hardworking, athletic, and otherwise in good health. There is a high clinical correlation between POTS and chronic fatigue syndrome. Although no single etiology for POTS has been found, the condition is thought to have a genetic predisposition, is often incited after a prolonged viral illness, and has a component of deconditioning. The recommended initial management is encouraging adequate fluid and salt intake, followed by the initiation of regular aerobic exercise combined with lower-extremity strength training, and then the use of β-blockers.
A 30-year-old male presents to the emergency department with a sensation of a racing heart. His history is significant for known Wolff-Parkinson-White syndrome (WPW). On examination he is alert and in no severe distress. His blood pressure is 130/70 mm Hg, pulse rate 220 beats/min, and oxygen saturation 96%. An EKG reveals a regular, wide-complex tachycardia with a rate of 220 beats/min. You determine that he is stable, the EKG is consistent with WPW, and pharmacologic conversion is a safe initial therapy. Which one of the following would be the treatment of choice?
*Procainamide* is usually the treatment of choice in these situations, although *amiodarone* may also be used. Adenosine, digoxin, and calcium channel antagonists act by blocking conduction through the atrioventricular (AV) node, which may increase the ventricular rate paradoxically, initiating ventricular fibrillation. *These agents should be avoided in Wolff-Parkinson-White syndrome*.
Which one of the following seafood poisonings requires more than just supportive treatment?
*Scombroid poisoning* is a pseudoallergic condition resulting from consumption of improperly stored scombroid fish such as tuna, mackerel, wahoo, and bonito. Nonscombroid varieties such as mahi-mahi, amberjack, sardines, and herring can also cause this problem. The poisoning is due to high levels of histamine and saurine resulting from bacterial catabolism of histidine. Symptoms occur within minutes to hours, and include flushing of the skin, oral paresthesias, pruritus, urticaria, nausea, vomiting, diarrhea, vertigo, headache, bronchospasm, dysphagia, tachycardia, and hypotension. Therapy should be the same as for allergic reactions and anaphylaxis, and will usually lead to resolution of symptoms within several hours.
An asymptomatic 35-year-old female asks about having a thyroid test performed because hypothyroidism runs in her family. You order the tests, which show a TSH level of 7.6μU/mL (N 0.4-5.1) and a free T4 level within the normal range. Which one of the following is most likely in this patient?
*Subclinical hypothyroidism*, defined as slightly elevated TSH (approximately 5-10 mIU/L) and normal levels of thyroid hormone (free T4 or free T3 ) in an asymptomatic patient. There is a low rate of progression to overt hypothyroidism manifested by symptoms, TSH levels >10 mIU/L, or reduced levels of thyroid hormone. Recent studies have shown that there is an increased risk for cardiovascular morbidity and mortality in those with subclinical hypothyroidism. However, treatment with thyroid replacement hormone did not seem to affect this risk. An alternative to treating the patient with medication at this time would be to retest her TSH annually, or sooner if she becomes symptomatic.
A 62-year-old male with a history of prostate cancer and well-controlled hypertension presents with severe osteoporosis. At 55 years of age he received prostate brachytherapy and androgen deprivation for his prostate cancer and has been disease-free since. He presently takes lisinopril (Prinivil, Zestril), 5 mg daily; alendronate (Fosamax), 70 mg weekly; calcium, 1000 mg daily; and vitamin D, 1200 units daily. He has never smoked, exercises five times a week, and maintains a healthy lifestyle. In spite of his lifestyle and the medications he takes, he continues to have severe osteoporosis on his yearly bone density tests. In addition to recommending fall precautions, which one of the following would you consider next to treat his osteoporosis?
*Teriparatide* (Forteo) is indicated for the treatment of severe osteoporosis, for patients with multiple osteoporosis risk factors, or for patients with failure of bisphosphonate therapy (SOR B). Therapy with teriparatide is currently limited to 2 years and is contraindicated in patients with a history of bone malignancy, Paget disease, hypercalcemia, or previous treatment with skeletal radiation. Its route of administration (subcutaneous) and high cost should be considered when prescribing teriparatide therapy. *Testosterone* therapy is contraindicated in patients with a history of prostate cancer. *Zoledronic acid* is a parenterally administered bisphosphonate and would not be appropriate in a patient who has already failed bisphosphonate therapy. Likewise, *raloxifene and calcitonin* are not indicated in patients with severe osteoporosis who have failed bisphosphonate therapy.
Which tinea infection in children always requires systemic antifungal therapy?
*Tinea capitis* requires systemic therapy to penetrate the affected hair shafts. Tinea cruris and tinea pedis rarely require systemic therapy. Extensive outbreaks of tinea corporis and tinea versicolor benefit from both oral and topical treatment (SOR A), but more localized infections require only topical treatment.
You have been treating a 43-year-old male for unipolar depression for 4 years. He has developed treatment-resistant depression, and despite having a good initial response to an SSRI, his symptoms are worsening. He has failed to improve despite escalated doses of multiple SSRIs and SNRIs. He is currently taking citalopram (Celexa), 60 mg daily. Of the following, the most effective adjunctive therapy would be augmentation with
*lithium bicarbonate*. Up to 1/3 of patients with depression will fail to respond to treatment with a single antidepressant, despite adequate dosing and an appropriate treatment interval. *Lithium, triiodothyronine (T3), and atypical antipsychotics* can all provide clinical improvement when used in conjunction with the ineffective antidepressant. The APA recommends a trial of lithium or low-dose T3 for patients who have an incomplete response to antidepressant therapy. A meta-analysis showed that a serum lithium level ≥0.5 mEq/L and a treatment duration of 2 weeks or greater resulted in a good response (SOR A). While thyroid supplementation as adjunctive therapy is effective, the recommended dosage is no higher than 50 μg/day (SOR B). Atypical antipsychotics can be used as add-on therapy, but are not as effective as lithium or T3 (SOR B). Anticonvulsant medications such as gabapentin have been shown to be effective in the management of bipolar affective disorder, but not as adjunctive therapy in the treatment of unipolar depression resistant to single-agent antidepressants.
What are the 4 pharmacological interventions for managing acute decompensated CHF?
1. Oxygen 2. Furosemide - diuretic with rapid bronchial vasodilation action 3. IV Nitroglycerin - reduces preload and afterload, reducing myocardial O2 demand 4. IV morphine sulfate - analgesic, anxiolytic, venodilator, arterial dilator
You are asked to perform a preoperative evaluation on a 55-year-old white female with type 2 diabetes mellitus prior to elective femoral-anterior tibial artery bypass surgery. She is unable to climb a flight of stairs or do heavy work around the house. She denies exertional chest pain, and is otherwise healthy. Based on current guidelines, which one of the following diagnostic studies would be appropriate prior to surgery because the results could alter the management of this patient?
A *dipyridamole-thallium scan*. Family physicians are often asked to perform a preoperative evaluation prior to noncardiac surgery. This requires an assessment of the perioperative cardiovascular risk of the procedure involved, the functional status of the patient, and clinical factors that can increase the risk, such as diabetes mellitus, stroke, renal insufficiency, compensated or prior heart failure, mild angina, or previous myocardial infarction. This patient is not undergoing emergency surgery, nor does she have an active cardiac condition; however, she is undergoing a high-risk procedure (>5% risk of perioperative myocardial infarction) with vascular surgery. As she cannot climb a flight of stairs or do heavy housework, her functional status is <4 METs, and she should be considered for further evaluation. The patient's diabetes is an additional clinical risk factor. With vascular surgery being planned, appropriate recommendations include proceeding with the surgery with heart rate control, or performing noninvasive testing if it will change the management of the patient. Coronary angiography is indicated if the noninvasive testing is abnormal. Pulmonary function studies are most useful in patients with underlying lung disease or those undergoing pulmonary resection. Hemoglobin A1c is a measure of long-term diabetic control and is not particularly useful perioperatively. Carotid angiography is not indicated in asymptomatic patients being considered for lower-extremity vascular procedures.
A 59-year-old male reports decreases in sexual desire and spontaneous erections, as well as reduced beard growth. The most appropriate test to screen for late-onset male hypogonadism is:
A *serum total testosterone level* is recommended as the initial screening test for late-onset male hypogonadism. Due to its high cost, a *free testosterone level* (a different lab value) is recommended only if the total testosterone level is borderline and abnormalities in sex hormone-binding globulin are suspected. Follow-up *LH and FSH levels* help to distinguish primary from secondary hypogonadism.
Which one of the following patients should be advised to take aspirin, 81 mg daily, for the primary prevention of stroke?
A 72-year-old female with no chronic medical conditions The USPSTF recommends the use of aspirin for: *men 45-79* and *women 55-79* when the potential benefit from a reduction in myocardial infarctions outweighs the potential harm from an increase in gastrointestinal hemorrhage (Grade A recommendation) The USPSTF recommends *against* the use of aspirin for stroke prevention in *women younger than 55* and for myocardial infarction prevention in *men younger than 45* (Grade D recommendation)
A 35-year-old African-American female with symptomatic uterine fibroids that are unresponsive to medical management prefers to avoid a hysterectomy. Which one of the following would be a reason for preferring myomectomy over fibroid embolization?
A desire for future pregnancy In the symptomatic patient with uterine fibroids unresponsive to medical therapy, *myomectomy* is recommended over fibroid embolization for patients who wish to become pregnant in the future. Uterine fibroid embolization requires a shorter hospitalization and less time off work. General anesthesia is not required, and a blood transfusion is unlikely to be needed. Uterine fibroids can recur or develop after either myomectomy or embolization.
A 55-year-old female presents to an urgent-care facility with a complaint of weakness of several weeks' duration. She has no other symptoms. She has been healthy except for a history of hypertension that has been difficult to control despite the use of hydrochlorothiazide, 25 mg daily; lisinopril (Prinivil, Zestril), 40 mg daily; amlodipine (Norvasc), 10 mg daily; and doxazosin (Cardura), 8 mg daily. On examination her blood pressure is 164/102 mm Hg, with the optic fundi showing grade 2 changes. She has normal pulses, a normal cardiac examination, and no abdominal bruits. A CBC is normal and a blood chemistry panel is also normal except for a serum potassium level of 3.1 mmol/L (N 3.5-5.5). What is the best way to confirm the most likely diagnosis in this patient?
A morning plasma aldosterone/renin ratio. Difficult-to-control hypertension has many possible causes, including nonadherence, alcohol use, NSAIDs, certain antidepressants, or sympathomimetics. Secondary hypertension can be caused by chronic kidney disease, obstructive sleep apnea, or *primary hyperaldosteronism*, as in this case. As many as 20% of patients referred to specialists for poorly controlled hypertension have primary hyperaldosteronism. It is more common in women and often is asymptomatic. A significant number of these individuals will not be hypokalemic, but those that do can present with nonspecific weakness. Screening can be done with a morning plasma aldosterone/renin ratio. If the ratio is *20 or more* and the aldosterone level is >15 ng/dL, then primary hyperaldosteronism is likely and referral for confirmatory testing should be considered.
A 4-year-old is brought to the emergency department with abdominal pain and is noted to have 3+ proteinuria on a dipstick. Three days later the pain has resolved spontaneously, and a repeat urinalysis in your office shows 2+ proteinuria with normal findings on microscopic examination. A metabolic panel, including creatinine and total protein, is also normal. Which one of the following would be most appropriate at this point?
A spot first morning urine protein/creatinine ratio. When proteinuria is noted on a dipstick and the history, examination, full urinalysis, and serum studies suggest no obvious underlying problem or renal insufficiency, a urine protein/creatinine ratio is recommended. This test correlates well with 24-hour urine protein, which is particularly difficult to collect in a younger patient. Renal ultrasonography is appropriate once renal insufficiency or nephritis is established. If pathogenic proteinuria is confirmed, an antinuclear antibody and/or complement panel may be indicated. A nephrology referral is not necessary until the presence of kidney disease or proteinuria from a cause other than benign postural proteinuria is confirmed.
A 60-year-old male is referred to you by his employer for management of his hypertension. He has been without primary care for several years due to a lapse in insurance coverage. During a recent employee health evaluation, he was noted to have a blood pressure of 170/95 mm Hg. He has a 20-year history of hypertension and suffered a small lacunar stroke 10 years ago. He has no other health problems and does not smoke or drink alcohol. A review of systems is negative except for minor residual weakness in his right upper extremity resulting from his remote stroke. His blood pressure is 168/98 mm Hg when initially measured by your nurse, and you obtain a similar reading during your examination. In addition to counseling him regarding lifestyle modifications, which one of the following is the most appropriate treatment for his hypertension?
A thiazide diuretic/ACE inhibitor combination This patient has stage 2 hypertension, and his history of stroke is a compelling indication to use specific classes of antihypertensives. For patients with a history of previous stroke, JNC-7 recommends using combination therapy with a diuretic and an ACE inhibitor to treat the hypertension, as this combination has been clinically shown to reduce the risk of recurrent stroke. Other classes of drugs have not been shown to be of benefit for secondary stroke prevention. Although blood pressure should not be lowered quickly in the setting of acute ischemic stroke, this patient is not having an acute stroke, so treatment of his hypertension is warranted.
In adults, the most common cause of right heart failure is:
Although myocarditis, pulmonic stenosis, and ventricular septal defects can be causes of right heart failure, *left heart failure* is the most common cause of right heart failure in adults.
A 48-year-old white female comes to see you because of abnormal vaginal bleeding. Her periods are lasting 3-5 days longer than usual, bleeding is heavier, and she has experienced some intermenstrual bleeding. Her physical examination is unremarkable, except for a parous cervix with dark blood at the os and in the vagina. She has no orthostatic hypotension, and her hemoglobin level is 11.5 g/dL. A pregnancy test is negative. Which one of the following is the most important next step in management?
An *endometrial biopsy*. A patient *>35 yo* with abnormal vaginal bleeding must have an endometrial assessment to exclude endometrial hyperplasia or cancer with endometrial biopsy. A laboratory evaluation for thyroid dysfunction or hemorrhagic diathesis is appropriate if no cancer is present on an endometrial biopsy and medical therapy fails to halt the bleeding.
A 28-year-old male recreational runner has a midshaft posteromedial tibial stress fracture. Although he can walk without pain, he cannot run without pain. The most appropriate treatment at this point includes which one of the following?
An air stirrup leg brace (*Aircast*) Midshaft posteromedial tibial stress fractures are common and are considered low risk. Management consists of relative rest from running and avoiding other activities that cause pain. Once usual daily activities are pain free, low-impact exercise can be initiated and followed by a gradual return to previous levels of running. A pneumatic stirrup leg brace has been found to be helpful during treatment (SOR C). Non-weight bearing is not necessary, as this patient can walk without pain. Casting is not recommended. Ultrasonic pulse therapy has helped fracture healing in some instances, but has not been shown to be beneficial in stress fractures.
Which one of the following is associated with vacuum-assisted delivery?
An increased incidence of shoulder dystocia Vacuum-assisted delivery is associated with higher rates of neonatal cephalhematoma and retinal hemorrhage compared with forceps delivery. A systematic review of 10 trials found that vacuum-assisted deliveries are associated with less maternal soft-tissue trauma when compared to forceps delivery. Compared with spontaneous vaginal delivery, the likelihood of a severe perineal laceration is increased in women who have vacuum-assisted delivery without episiotomy, and the odds are even higher in vacuum-assisted delivery with episiotomy. Operative vaginal delivery is a risk factor for shoulder dystocia, which is more common with vacuum-assisted delivery than with forceps delivery.
An 18-year-old male presents with a sore throat, adenopathy, and fatigue. He has no evidence of airway compromise. A heterophil antibody test is positive for infectious mononucleosis. Appropriate management includes which one of the following?
Avoidance of contact sports Infectious mononucleosis presents most commonly with a sore throat, fatigue, myalgias, and lymphadenopathy, and is most prevalent between 10 and 30 years of age. Both an atypical lymphocytosis and a positive heterophil antibody test support the diagnosis, although false-negative heterophil testing is common early in the disease course. The cornerstone of treatment for mononucleosis is supportive, including hydration, NSAIDs, and throat sprays or lozenges. In general, corticosteroids do not have a significant effect on the clinical course of infectious mononucleosis, and they should not be used routinely unless the patient has evidence of acute airway obstruction. Antihistamines are also not recommended as routine treatment for mononucleosis. The use of acyclovir has shown no consistent or significant benefit, and antiviral drugs are not recommended. There is also no evidence to support bed rest as an effective management strategy for mononucleosis. Given the evidence from other disease states, bed rest may actually be harmful. Although most patients will not have a palpably enlarged spleen on examination, it is likely that all, or nearly all, patients with mononucleosis have splenomegaly. This was demonstrated in a small study in which 100% of patients hospitalized for mononucleosis had an enlarged spleen by ultrasound examination, whereas only 17% of patients with splenomegaly have a palpable spleen. Patients should be advised to avoid contact- or collision-type activities for 3-4 weeks because of the increased risk of rupture.
A primigravida at 38 weeks gestation is concerned that her fetus is getting too large and wants to know what interventions could prevent complications from a large baby. On examination her uterine fundus measures 41 cm from the pubic symphysis. Ultrasonography is performed and an estimated fetal weight of 4000 g (8 lb 13 oz) is reported. Which one of the following management options is supported by the best evidence?
Awaiting spontaneous labor. This estimated fetal weight is at the 90th percentile for a term fetus. Unfortunately, the accuracy of fetal weight estimates declines as pregnancy proceeds, and the actual size may be as much as 15% different from the estimate. Delivery of a large infant results in shoulder dystocia more often than delivery of a smaller infant, but most large infants are delivered without complications. Intuitively, it would seem logical to induce labor when the fetus seems to be getting large, but this intervention has been studied in controlled trials and the only difference in outcome was an increase in the cesarean rate for women who underwent elective induction for this indication. Recently, there has been an increase in requests from patients to have an elective cesarean section near term to avoid the risks of labor, including pain, shoulder dystocia, and pelvic relaxation. The American Congress of Obstetricians and Gynecologists (ACOG) recommends consideration of cesarean delivery without a trial of labor if the estimated fetal weight is 4500 g in a mother with diabetes mellitus, or 5000 g in the absence of diabetes. Even at that size, there is not adequate data to show that cesarean section is preferable to a trial of labor. Frequent ultrasonography is often performed to reduce anxiety for both patient and physician, but the problem of accuracy of weight estimates remains an issue even with repeated scans at term.
One of your patients has been diagnosed with monoclonal gammopathy of undetermined significance (MGUS). Which one of the following is used to determine whether his condition has progressed to multiple myeloma?
Evidence of end-organ damage The diagnosis of multiple myeloma is based on evidence of myeloma-related end-organ impairment in the presence of M protein, monoclonal plasma cells, or both. This evidence may include *hypercalcemia, renal failure, anemia, or skeletal lesions*. Monoclonal gammopathy of undetermined significance does not progress steadily to multiple myeloma. There is a stable 1% annual risk of progression.
Because of safety concerns, which one of the following asthma medications should be used only as additive therapy and not as monotherapy?
Because of the risk of asthma exacerbation or asthma-related death, the FDA has added a warning against the use of *long-acting β2-agonists* (Salmeterol) as monotherapy. Inhaled corticosteroids, leukotriene-receptor antagonists, short-acting β2-agonists, and mast-cell stabilizers are approved and accepted for both monotherapy and combination therapy in the management of asthma (SOR A).
An 82-year-old resident of a local nursing home is brought to your clinic with fever, difficulty breathing, and a cough productive of purulent sputum. The patient is found to have an oxygen saturation of 86% on room air and a chest radiograph shows a new infiltrate. A decision is made to hospitalize the patient. Which one of the following intravenous antibiotic regimens would be most appropriate for this patient?
Ceftazidime, levofloxacin, and vancomycin Nursing home-acquired pneumonia should be suspected in patients with a new infiltrate on a chest radiograph if it is associated with a fever, leukocytosis, purulent sputum, or hypoxia. They require empiric coverage for MRSA and Pseudomonas with an antipseudomonal cephalosporin such as *cefepime or ceftazidime*, an antipseudomonal carbapenem such as *imipenem or meropenem*, or an extended-spectrum β-lactam/β-lactamase inhibitor such as *piperacillin/tazobactam*, PLUS an antipseudomonal fluoroquinolone such as *levofloxacin or ciprofloxacin*, or an aminoglycoside such as *gentamicin, tobramycin, or amikacin*, PLUS an anti-MRSA agent (*vancomycin or linezolid*). Ceftriaxone and azithromycin or levofloxacin alone would be reasonable treatment options for a patient with nursing home-acquired pneumonia who does not require hospitalization.
A 21-year-old African-American female has been confused and delirious for 2 days. She has no significant past medical history, and she is taking no medications. She recently returned from a missionary trip to Southeast Asia. During your initial examination in the emergency department, she has several convulsions and rapidly becomes comatose. Her temperature is 37.9°C (100.3°F) and her blood pressure is 80/50 mm Hg. A neurologic examination shows no signs of meningeal irritation and a cranial nerve evaluation is normal. There is a mild, bilateral, symmetric increase in deep tendon reflexes. All other physical examination findings are normal. Laboratory Findings Hemoglobin........................... 7.0 g/dL (N 12.0-16.0) Hematocrit............................ 20% (N 36-46) WBCs.. . . . . . . . . . . . . . . . . . . . . . . . . . . . . . . 6500/mm3 (N 4300-10,800) Platelets. ............................. 450,000/mm3 (N 150,000-350,000) Serum bilirubin Total............................... 5.0 mg/dL (N 0.3-1.1) Direct.............................. 1.0 mg/dL (N 0.1-0.4) The urine is dark red and positive for hemoglobin. CT of the brain shows neither bleeding nor infarction. The most likely diagnosis is:
Clinical clues to the diagnosis of *malaria* in this case include an appropriately targeted recent travel history, a prodrome of delirium or erratic behavior, unarousable coma following a generalized convulsion, fever, and a lack of focal neurologic signs in the presence of a diffuse, symmetric encephalopathy. The peripheral blood smear will show a normochromic, normocytic anemia with Plasmodium falciparum trophozoites and schizonts involving erythrocytes, diagnostic of cerebral malaria. Treatment of this true medical emergency is *IV quinidine gluconate*. *Vitamin B12 deficiency* is a predominantly peripheral neuropathy seen in older adults. *Ehrlichiosis* (tick borne bacterial infection) causes thrombocytopenia but not hemolytic anemia. *Sickle cell disease* presents with painful vaso-occlusive crises in multiple organs. Coma is rare.
A 44-year-old female who suffers from obstructive sleep apnea complains of gradual swelling in her legs over the last several weeks. Her vital signs include a BMI of 44.1 kg/m2 , a respiratory rate of 12/min, a blood pressure of 120/78 mm Hg, and an O 2 saturation of 86% on room air. An EKG and a chest radiograph are normal. Pulmonary function testing shows a restrictive pattern with no signs of abnormal diffusion. Abnormal blood tests include only a significantly elevated bicarbonate level. Which one of the following treatments is most likely to reduce this patient's mortality rate?
Continuous or bilevel positive airway pressure (CPAP or Bi-PAP) This patient has obesity-hypoventilation syndrome, often referred to as Pickwickian syndrome. These patients are obese (BMI >30 kg/m 2 ), have sleep apnea, and suffer from chronic daytime hypoxia andcarbon dioxide retention. They are at increased risk for significant respiratory failure and death compared to patients with otherwise similar demographics. Treatment consists of nighttime positive airway pressure in the form of continuous (CPAP) or bi-level (BiPAP) devices, as indicated by sleep testing. The more hours per day that patients can use this therapy, the less carbon dioxide retention and less daytime hypoxia will ensue. Several small studies suggest that the increased mortality risk from obesity-hypoventilation syndrome can be decreased by adhering to this therapy. The use of daytime oxygen can improve oxygenation, but is not considered adequate to restore the chronic low respiratory drive that is characteristic of this condition.
A 68-year-old African-American female with primary hypothyroidism is taking levothyroxine (Synthroid), 125 μg/day. Her TSH level is 0.2μU/mL (N 0.5-5.0). She has no symptoms of either hypothyroidism or hyperthyroidism. Which one of the following would be most appropriate at this point?
Decreasing the levothyroxine dosage. Because of the precise relationship between circulating thyroid hormone and pituitary TSH secretion, measurement of serum TSH is essential in the management of patients receiving levothyroxine therapy. In a patient receiving levothyroxine, a low TSH level usually indicates overreplacement. If this occurs, the dosage should be reduced slightly and the TSH level *repeated in 2-3 months' time*. There is no need to discontinue therapy in this situation, and repeating the TSH level in 2 weeks would not be helpful. A free T4 level would also be unnecessary, since it is not as sensitive as a TSH level for detecting mild states of excess thyroid hormone
A 62-year-old male has been taking omeprazole (Prilosec) for over a year for gastroesophageal reflux disease. He is asymptomatic and has had no problems tolerating the drug, but asks you about potential side effects, as well as the benefits of continuing therapy. It would be most accurate to tell him that omeprazole therapy is associated with which one of the following?
Decreased vitamin B12 absorption. Although proton pump inhibitors are the most effective treatment for patients with asymptomatic gastroesophageal reflux disease, there are several potential problems with prolonged therapy. Omeprazole is associated with an increased risk of *community-acquired pneumonia* and *Clostridium difficile colitis*. Omeprazole has also been shown to acutely decrease the absorption of *vitamin B12* , and it decreases *calcium* absorption, leading to an increased risk of *hip fracture*.
What is the most common cause of erythema multiforme, accounting for more than 50% of cases?
Erythema multiforme usually occurs in adults 20-40 years of age, although it can occur in patients of all ages. Herpes simplex virus (*HSV*) is the most commonly identified cause of this hypersensitivity reaction, accounting for more than 50% of cases.
The Valsalva maneuver will typically cause the intensity of a systolic murmur to increase in patients with which one of the following conditions?
Hypertrophic obstructive cardiomyopathy Valsalva maneuver → *↓venous return to the heart* → ↓cardiac output ↓cardiac output → most murmurs to ↓length and ↓intensity. The murmur of hypertrophic obstructive cardiomyopathy, however, ↑loudness. The murmur of mitral valve prolapse becomes longer, and may also become louder.
A 40-year-old female with chronic plaque psoriasis requests topical treatment. Which one of the following topical therapies would be most effective and have the fewest adverse effects?
High-potency corticosteroids. Chronic plaque psoriasis is the most common type of psoriasis and is characterized by redness, thickness, and scaling. A variety of treatments were found to be more effective than placebo, but the best results were produced by topical vitamin D analogues and topical corticosteroids. Vitamin D and high-potency corticosteroids were equally effective when compared head to head, but the corticosteroids produced fewer local reactions (SOR A).
A 25-year-old medical student reads about the benefits of moderate alcohol consumption on lipid levels and begins to drink 5 ounces of red wine a day, adding 100 calories to his diet. Assuming that his diet and exercise levels stay the same, what effect will the additional 3000 calories a month have on his body weight over the next 10 years?
His weight will increase slightly then stabilize There is *not* a direct relation between daily calorie consumption and weight. An adult male consuming an extra 100 calories a day above his caloric need will not continue to gain weight indefinitely; rather, his weight will increase to a certain point and then become constant. Fat must be fed, and maintaining the newly created tissue requires an increase in caloric expenditure. An extra 100 calories a day will result in a *weight gain of approximately 5 kg*, which will then be maintained.
A 45-year-old female presents with a 3-month history of hoarseness that is not improving. She works as a high-school teacher. The most appropriate management at this time would be:
Hoarseness most commonly affects teachers and older adults. The cause is usually benign, but extended symptoms or certain risk factors should prompt evaluation; specifically, *laryngoscopy* is recommended when hoarseness does not resolve within 3 months or when a serious underlying cause is suspected antireflux medications should not be prescribed for patients with hoarseness without reflux symptoms (SOR C). Antibiotics should not be used, as the condition is usually caused by acute laryngitis or an upper respiratory infection, and these are most likely to be viral. Inhaled corticosteroids are a common cause of hoarseness. Voice therapy should be reserved for patients who have undergone laryngoscopy first (SOR A).
A 64-year-old African-American male presents with persistent pleuritic pain. The patient does not feel well in general and has had a low-grade fever of around 100°F (38°C). His medications include simvastatin (Zocor), lisinopril (Prinivil, Zestril), low-dose aspirin, spironolactone (Aldactone), furosemide (Lasix), isosorbide mononitrate (Imdur), hydralazine, carvedilol (Coreg), and nitroglycerin as needed. A chest radiograph is normal and does not demonstrate a pneumothorax. Further evaluation rules out pulmonary embolus, pneumonia, and myocardial infarction. A diagnosis of pleurisy is made. Which one of the patient's medications could be related to this condition?
Hydralazine Drug-induced pleuritis is one cause of pleurisy. Several drugs are associated with drug-induced pleural disease or drug-induced lupus pleuritis. Drugs that may cause lupus pleuritis include *hydralazine, procainamide, and quinidine*. Other drugs known to cause pleural disease include *amiodarone, bleomycin, bromocriptine, cyclophosphamide, methotrexate, minoxidil, and mitomycin*.
A 65-year-old Hispanic male with known metastatic lung cancer is hospitalized because of decreased appetite, lethargy, and confusion of 2 weeks' duration. Laboratory evaluation reveals the following: Serum calcium......................... 15.8 mg/dL (N 8.4-10.0) Serum phosphorus...................... 3.9 mg/dL (N 2.6-4.2) Serum creatinine. ...................... 1.1 mg/dL (N 0.7-1.3) Total serum protein..................... 5.0 g/dL (N 6.0-8.0) Albumin.............................. 3.1 g/dL (N 3.7-4.8) Which one of the following is the most appropriate *initial* management?
IV normal saline; the initial management of *hypercalcemia of malignancy* includes *fluid replacement with normal saline* to correct the volume depletion that is invariably present and to enhance renal calcium excretion. The use of loop diuretics such as furosemide should be *restricted* to patients in danger of fluid overload, since these drugs can aggravate volume depletion and are not very effective alone in promoting renal calcium excretion. Although intravenous *pamidronate* has become the mainstay of treatment for the hypercalcemia of malignancy, it is considered *only after* the hypercalcemic patient has been rendered euvolemic by saline repletion. The same is true for the other calcium-lowering agents listed.
You see a 90-year-old male with a 5-year history of progressive hearing loss. The most common type of hearing loss at this age affects:
In the geriatric population, *presbycusis* is the most common cause of hearing loss. Patients typically have the most difficulty hearing *higher-frequency sounds* such as consonants. Lower-frequency sounds such as vowels are preserved.
Which one of the following is most characteristic of patellofemoral pain syndrome in adolescent females?
Inadequate hip abductor strength Patellofemoral pain syndrome is a common overuse injury observed in adolescent girls. The condition is characterized by anterior knee pain associated with activity. The pain is exacerbated by going up or down stairs or running in hilly terrain. It is associated with inadequate hip abductor and core strength; therefore, a prescription for a rehabilitation program is recommended. Surgical intervention is rarely required.
Which one of the following is associated with the use of percutaneous endoscopic gastrostomy (PEG) tubes?
Increased use of restraints When a patient or nursing-home resident is losing weight or has suffered an acute change in the ability to perform activities of daily living, a decision must be made as to whether or not to place a PEG tube to provide artificial nutrition. Studies have shown that PEG tubes *do not improve nutritional status or quality of life* for residents with dementia, nor do they decrease the risk of aspiration pneumonia, although aspiration risk may possibly be decreased if the feeding tube is placed below the gastroduodenal junction (SOR B). Feeding tubes can also *cause discomfort and agitation*, leading to an increased use of restraints (SOR B).
A 12-month-old white female whom you have seen regularly for all of her scheduled well child care is found to have a hemoglobin level of 9.0 g/dL (N for age 10.5-13.5). She started whole milk at 9 months of age. She appears healthy otherwise and has no family history of anemia. A CBC reveals a mild microcytic, hypochromic anemia with RBC poikilocytosis, but is otherwise normal. The RBC distribution width is also elevated. Of the following, the most appropriate next step would be to:
Iron deficiency is almost certainly the diagnosis in this child. The patient's response to a *therapeutic trial of oral iron* would be most helpful in establishing the diagnosis. Additional tests might be necessary if there is no response.
A 27-year-old female presents to the emergency department with a complaint of bloody diarrhea and abdominal cramping. A few days ago she ate a rare hamburger at a birthday party for her 4-year-old son. He ate hot dogs instead, and has not been ill. A stool specimen is positive for Escherichia coli O:157. Which one of the following should you do next?
Monitor her renal function. Escherichia coli O:157 is an increasingly common cause of serious gastrointestinal illness. The usual source is undercooked beef. The child is at risk, since at least 20% of cases result from secondary spread. Transmission is frequent in children's day-care facilities and nurseries. Some cases are asymptomatic, but the great majority are symptomatic, and patients present with bloody diarrhea. This patient has a 10%-15% risk of developing *hemolytic uremic syndrome* secondary to her E. coli O:157 infection, making close monitoring of renal function essential.
As the medical review officer for a local business, you are required to interpret urine drug tests. Assuming the sample was properly collected and handled, which one of the following test results is consistent with the history provided and should be reported as a negative test?
Morphine identified in an employee taking a prescribed cough medicine containing codeine Thresholds are set to preclude the possibility that secondary contact with smoke, ingestion of poppy seeds, or similar exposures will result in an undeserved positive urine drug screen report. Other findings, such as the presence of behavioral or physical evidence of unauthorized use of opiates, may also factor into the final report. When a properly collected, acceptable specimen is found to contain drugs or metabolites that would be expected based on a review of confirmed prescribed use of medications, the test is reported as negative. Morphine is a metabolite of codeine that may be found in the urine of someone taking a codeine-containing medication; morphine is not a metabolite of methadone. Oxazepam is a metabolite of diazepam (valium) but the reverse is not true. Tetrahydrocannabinol would not be found in the urine as a result of tramadol use.
Breastfeeding a full-term, healthy infant is contraindicated when which one of the following maternal conditions is present?
Mothers with *active herpes simplex lesions* on a breast should not feed their infant from the infected breast, but may do so from the other breast if it is not infected. Breastfeeding is also contraindicated in the presence of *active maternal tuberculosis*, and following administration or use of *radioactive isotopes, chemotherapeutic agents, "recreational" drugs, or certain prescription drugs*. Breastfeeding provides such optimal nutrition for an infant that the benefits still far outweigh the risks even when the mother smokes tobacco, tests positive for hepatitis B or C virus, or develops a simple undifferentiated fever. Maternal seropositivity to cytomegalovirus (CMV) is not considered a contraindication except when it has a recent onset or in mothers of low birthweight infants. When present, the CMV load can be substantially reduced by freezing and pasteurization of the milk. All patients who smoke should be strongly encouraged to discontinue use of tobacco, particularly in the presence of infants, but smoking is not a contraindication to breastfeeding.
A 25-year-old male presents to your office with a 1-week history of neck pain with radiation to the left hand, along with intermittent numbness and tingling in the left arm. His history is negative for injury, fever, or lower extremity symptoms. Extension and rotation of the neck to the left while pressing down on the head (Spurling's maneuver) exacerbates the symptoms. His examination is otherwise normal. Cervical radiographs are negative. Which one of the following would be most appropriate at this point?
NSAIDs for pain relief. Patients who present with acute cervical radiculopathy and normal radiographs can be treated conservatively. The vast majority of patients with cervical radiculopathy improve without surgery. Of the interventions listed, NSAIDs are the initial treatment of choice. Tricyclic antidepressants, as well as tramadol and venlafaxine, have been shown to help with chronic neuropathic pain. Cervical MRI is not indicated unless there are progressive neurologic defects or red flags such as fever or myelopathy. Likewise, referral to a subspecialist should be reserved for patients who have persistent pain after 6-8 weeks of conservative management and for those with signs of instability. Cervical corticosteroid injections have been found to be helpful in the management of cervical radiculopathy, but should not be administered before MRI is performed (SOR C).
A 15-month-old male is brought to your office 3 hours after the onset of an increased respiratory rate and wheezing. He has an occasional cough and no rhinorrhea. His immunizations are up to date and he attends day care regularly. His temperature is 38.2°C (100.8°F), respiratory rate 42/min, and pulse rate 118 beats/min. The child is sitting quietly on his mother's lap. His oxygen saturation is 94% on room air. On examination you note *inspiratory crackles in the left lower lung field*. The child appears to be well hydrated and the remainder of the examination, including an HEENT examination, is normal. Nebulized albuterol (AccuNeb) is administered and no improvement is noted. Which one of the following would be most appropriate in the management of this patient?
Oral high-dose *amoxicillin* (90 mg/kg/day), with close *outpatient* follow-up. The diagnosis of *community-acquired pneumonia* is mostly based on the history and physical examination. PNA should be suspected in any child with fever, cyanosis, and any abnormal respiratory finding in the history or physical examination. Children under 2 years of age who are in day care are at higher risk for developing CAP. Lab tests are rarely helpful in differentiating viral vs bacterial etiologies and should *not* be routinely performed. Outpatient antibiotics are appropriate *if* the child does not have a toxic appearance, hypoxemia, signs of respiratory distress, or dehydration. Streptococcus pneumoniae is one of the most common etiologies in this age group, and high-dose amoxicillin is the drug of choice.
A 40-year-old female comes to your office with a 1-month history of right heel pain that she describes as sharp, searing, and severe. The pain is worst when she first bears weight on the foot after prolonged sitting and when she gets out of bed in the morning. It gets better with continued walking, but worsens at the end of the day. She does not exercise except for being on her feet all day in the hospital where she works as a floor nurse. She denies any history of trauma. An examination reveals point tenderness to palpation on the plantar surface of the heel at the medial calcaneal tuberosity. Which one of the following should you recommend as first-line treatment?
Over-the-counter heel inserts. *Plantar fasciitis* is a common cause of heel pain. It may be unilateral or bilateral, and the etiology is unknown, although it is thought to be due to cumulative overload stress. While it may be associated with obesity or overuse, it may also occur in active or inactive patients of all ages. Typically the pain is located in the plantar surface of the heel and is worst when the patient first stands up when getting out of bed in the morning (first step phenomenon) or after prolonged sitting. The pain may then improve after the patient walks around, only to worsen after prolonged walking. The diagnosis is made by history and physical examination. Typical findings include point tenderness to palpation on the plantar surface of the heel at the medial calcaneal tuberosity where the calcaneal aponeurosis inserts. Radiographs are not necessary unless there is a history of trauma or if the diagnosis is unclear. The condition may last for months or years, and resolves in most patients over time with or without specific therapy. One long-term follow-up study showed that 80% of patients had complete resolution of their pain after 4 years. Treatments with limited (level 2) evidence of effectiveness include off-the-shelf insoles, custom-made insoles, stretching of the plantar fascia, corticosteroid iontophoresis, custom-made night splints, and surgery (for those who have failed conservative therapy). NSAIDs and ice, although not independently studied for plantar fasciitis, are included in most studies of other treatments, and are reasonable adjuncts to first-line therapy. Magnetic insoles and extracorporeal shockwave therapy are ineffective in treating plantar fasciitis. Due to their expense, custom-made insoles, custom-made night splints, and corticosteroid iontophoresis should be reserved as second-line treatments for patients who fail first-line treatment. Surgery may be offered if more conservative therapies fail. Corticosteroid injection may have a short-term benefit at 1 month, but is no better than other treatments at 6 months and carries a risk of plantar fascia rupture.
A 52-year-old male with stable coronary artery disease and controlled hypertension sees you for a routine visit and asks for advice regarding prevention of altitude illness for his upcoming trip to Bhutan to celebrate his anniversary. His medical chart indicates that he had a reaction to a sulfa drug in the past. Which one of the following would be most appropriate?
Prescribe *dexamethasone*. Altitude illness is common, affecting 25%-85% of travelers to high altitudes. The most common manifestation is acute mountain sickness, heralded by malaise and headache. Risk factors include young age, residence at a low altitude, rapid ascent, strenuous physical exertion, and a previous history of altitude illness. However, activity restriction is not necessary for patients with coronary artery disease who are traveling to high altitudes (SOR C). *Ginkgo biloba* has been evaluated for both prevention and treatment of acute mountain sickness and high-altitude cerebral edema, and it is *not* recommended. *Acetazolamide* is an effective prophylactic agent (SOR B), but is contraindicated in patients with a sulfa allergy. If used, it should be started a minimum of one day before ascent and continued until the patient acclimatizes at the highest planned elevation. *Dexamethasone* is an effective prophylactic and treatment agent (SOR B), and it is not contraindicated for those with a sulfa allergy. It would be the best option for this patient.
Which one of the following is most consistent with obsessive-compulsive disorder in adults?
Recognition by the patient that the obsessions or compulsions are excessive or unreasonable The impulses of OCD are not related to excessive worry about one's problems, and the patient recognizes that they are the product of his or her own mind (not "inserted"). In addition, the patient experiences marked distress because of the impulses. Full remission is rare, but treatment can provide significant relief.
A 70-year-old African-American male undergoes routine sigmoidoscopy. He has a long history of constipation, hypertension, and diet-controlled type 2 diabetes mellitus. The examination reveals brown to black leopard spotting of the colonic mucosa. You would now do what?
Review his medications This patient has *melanosis coli*, which is a benign condition resulting from abuse of anthraquinone laxatives such as cascara, senna, or aloe. The condition resolves with discontinuation of the medication.
A 52-year-old male has had a chronic course of multiple vague and exaggerated symptoms for which no cause has been found despite extensive testing. Which one of the following is the most effective management approach for this patient?
Schedule the patient for regular appointments every 2-4 weeks The management of somatizing patients can be difficult. One strategy that has been shown to be effective is to schedule regular office visits so that the patient does not need to develop new symptoms in order to receive medical attention. Regular visits have been shown to significantly reduce the cost and chaos of caring for patients with somatization disorder and to help progressively diminish emergency visits and telephone calls. In addition, it is important to describe the patient's diagnosis with compassion and avoid suggesting that it's "all in your head."
What psych drugs has the best evidence that it is safe for use in pregnancy?
Studies have shown *no* significant risk of congenital anomalies from SSRI (*Fluoxetine*) use in pregnancy, except for *paroxetine*, which is a category D medication and should be avoided in pregnant women due to risk of congenital cardiac malformations from first-trimester exposure. *Lithium* is known to be teratogenic. Benzodiazepines such as *alprazolam* are controversial due to a possible link to cleft lip/palate. *Bupropion* has not been studied extensively for use in pregnancy, and in one published study of 136 patients it was linked to an increased risk of spontaneous abortion.
A 12-year-old male presents with left hip pain. He is overweight and recently started playing tennis to lose weight. He says the pain started gradually after his last tennis game, but he does not recall any injury. He is walking with a limp. On examination he is afebrile and has limited internal rotation of the left hip. What is the most likely cause of the hip pain?
Slipped capital femoral epiphysis Slipped capital femoral epiphysis is the most common hip disorder in this patient's age group. It usually occurs between the ages of 8 and 15 and is more common in boys and overweight or obese children. It presents with limping and pain, and limited internal rotation of the hip is noted on physical examination. Septic arthritis would typically present with a fever. Juvenile rheumatoid arthritis, transient synovitis, and Legg-Calvé-Perthes disease are more common in younger children.
One week after a complete and adequate baseline screening colonoscopy, a 51-year-old female with no history of previous health problems visits you to review the pathology report on the biopsy specimen obtained from the solitary 8-mm polyp discovered in her sigmoid colon. The report confirms that this was a hyperplastic polyp. Her family history is negative for colon cancer. Which one of the following is the most appropriate interval for follow-up colonoscopy in this patient?
Small (<10 mm) hyperplastic polyps found in the rectum or sigmoid colon are *not* neoplastic and are included in the same *low-risk* cohort as those who have an unremarkable colonoscopy. For patients at low risk the recommended interval between screening colonoscopies is *10 years*. Reductions in this interval are recommended for patients with one or two small *tubular adenomas* (5-10 years) or those with three or more tubular adenomas (3 years)
A 58-year-old white male comes to your office for follow-up after a recent bout of acute bronchitis. He reports having a productive cough for several months. He gets breathless with exertion and notes that every time he gets a cold it "goes into my chest and lingers for months." He has been smoking for 30 years. A physical examination is negative except for scattered rhonchi. A chest radiograph done 4 months ago at an urgent care visit was negative except for hyperinflation and flattened diaphragms. Which one of the following would be best for making the diagnosis?
Spirometry It is important to distinguish between COPD and asthma because of the differences in treatment. Patients with COPD are usually in their sixties when the diagnosis is made. Symptoms of chronic cough (sometimes for months or years), dyspnea, or sputum production are often not reported because the patient may attribute them to smoking, aging, or poor physical condition. Spirometry is the best test for the diagnosis of COPD. The pressure of outflow obstruction that is not fully reversible is demonstrated by postbronchodilator spirometry showing an FEV /FVC ratio of 70% or less.
An 81-year-old African-American female complains of increasing fatigue over the past several months. She has also noticed that her skin and hair feel dry and that she often feels cold. She also complains of intermittent swallowing difficulties. Her past medical history is significant for long-standing coronary artery disease, for which she takes metoprolol (Lopressor). Her physical examination is normal except for a resting pulse rate of 56 beats/min, dry skin, brittle hair, and a slow relaxation phase of the deep tendon reflexes. Her serum TSH level is 63.2 μU/mL (N 0.5-5.0). Which one of the following should you do now?
Start levothyroxine (Synthroid). Autoimmune hypothyroidism is common in elderly women. Symptoms often include fatigue, bradycardia, dry skin, brittle hair, and a prolonged relaxation phase of the deep tendon reflexes. While replacement therapy with levothyroxine is indicated, care must be taken in the elderly, particularly in those with coronary artery disease, to replace the deficit slowly. Levothyroxine replacement should begin at 25μg daily for 6 weeks, with the dosage increased in 25-μg increments as needed, based on TSH levels. Rapid replacement of thyroid hormone can increase the metabolic rate, and therefore myocardial oxygen demand, too quickly. This can precipitate complications of coronary artery disease such as atrial fibrillation, angina, and myocardial infarction. Stopping a β-blocker in this setting is likely to increase the risk.
A nursing-home resident is hospitalized, and shortly before she is to be discharged she develops a skin ulcer, which proves to be infected with methicillin-resistant Staphylococcus aureus (MRSA). Which one of the following is most important in terms of infection control when she returns to the nursing home?
Strict handwashing practices by all staff, visitors, and residents Barrier precautions for wounds and medical devices should also be initiated. Surveillance cultures are not warranted.
A 25-year-old male presents to your office for evaluation of pain in the right index finger that has been present for the past 4 days. The pain has been getting progressively worse. On examination the finger is swollen and held in a flexed position. The pain increases with passive extension of the finger, and there is tenderness to palpation from the tip of the finger into the palm. Which one of the following is the most appropriate management of this patient?
Surgical drainage and antibiotic This patient has *pyogenic tenosynovitis*. When early tenosynovitis (within 48 hours of onset) is suspected, treatment with antibiotics and splinting may prevent the spread of the infection. However, this patient's infection is no longer in the early stages and is more severe, so it requires surgical drainage and antibiotics. A delay in treatment of these infections can lead to ischemia of the tendons and damage to the flexor tendon and sheath. This can lead to impaired function of the finger. Needle aspiration would not adequately drain the infection. Antiviral medication would not be appropriate, as this is a bacterial infection. Corticosteroid injections are contraindicated in the presence of infection.
With regard to the cardiovascular system, activation of the sympathetic branch of the autonomic nervous system will cause a decrease in which one of the following?
The P-R interval. The sympathetic nervous system acts as a positive chronotropic (increases heart rate) and inotropic (increases contractility) agent. This additional work by the heart will increase metabolic demand and coronary flow rate. The increased heart rate will decrease the time intervals between electrical events shown on an EKG.
Which one of the following screening practices is recommended for the adolescent population by the U.S. Preventive Services Task Force?
The U.S. Preventive Services Task Force recommends *screening for Chlamydia infection* in all sexually active, nonpregnant young women under the age of 25 (grade B recommendation). Papanicolaou testing is recommended starting at 21 years of age. Testicular cancer screening, whether by self-examination or as part of the physical examination, is not recommended. Scoliosis screening for asymptomatic adolescents is also not recommended. There is insufficient evidence to recommend for or against lipid screening.
A 21-year-old sexually active female presents with acute pelvic pain of several days' duration. A pelvic examination reveals right-sided tenderness and a general fullness in that area. In addition to laboratory testing, you decide to order an imaging study. Which one of the following is the best choice at this time?
The best initial imaging study for acute pelvic pain in women is *transvaginal ultrasonography* (SOR C). This provides the greatest level of detail regarding the uterus and adnexae, superior to transcutaneous ultrasonography. CT of the abdomen/pelvis and hysterosalpingography may be indicated eventually in some patients with pelvic pain, but they are not the initial studies of choice. Hysteroscopy is not routinely used in the evaluation of pelvic pain.
What symptoms are typical of polymyalgia rheumatica?
The clinical hallmarks of polymyalgia rheumatica are *pain and stiffness in the shoulder and pelvic girdle* in women >50. Many patients have a normal ESR Polymyalgia rheumatica can have a variety of systemic symptoms. *Fever* is common, with temperatures as high as 39°C (102°F) along with *night sweats*. Additional symptoms include *depression, fatigue, malaise, anorexia, and weight loss*. Corticosteroids are the mainstay of therapy for polymyalgia rheumatica. Typically, a *dramatic response is seen within 48-72 hours*.
The mother of a 16-year-old male brings him to your office stating that she wants to find out if he has Crohn's disease. She says that both she and the child's aunt were diagnosed with this condition by another physician with "blood tests." The son tells you that for the past several years his stool is intermittently loose and he has up to three bowel movements in a day. He says he does not have fever, pain, hematochezia, weight loss, or any extraintestinal symptoms. A physical examination is normal. Which one of the following would be the most appropriate preliminary testing?
The diagnosis of inflammatory bowel disease (IBD) can be elusive but relies primarily on the patient history, laboratory findings, and endoscopy (or double-contrast radiographs if endoscopy is not available). Endoscopy is usually reserved for patients with more severe symptoms or in whom preliminary testing shows the potential for significant inflammation. It is recommended that this preliminary evaluation include a *WBC count, platelet count, potassium level, and erythrocyte sedimentation rate*. Patients who have minimal symptoms and normal preliminary testing likely do not have a significant case of IBD. Plain radiographs and CT of the abdomen may help rule out other etiologies but are not considered adequate to diagnose or exclude IBD. Panels of serologic blood tests have recently been developed and are being assessed as to their place in evaluating patients who may have IBD. However, this testing is expensive, lacks sufficient predictive value, and has yet to prove its utility compared to standard testing.
You are preparing to evaluate a patient in the emergency department. A BNP level was ordered by the physician from the previous shift who handed the patient over to you. The level is reported as 459 pg/mL. You have not yet interviewed or examined the patient. Based on the information you have at this point, which one of the following is true regarding this patient?
The patient's diagnosis is uncertain Levels of natriuretic peptides such as BNP lack the specificity necessary to function as absolute indicators of acute heart failure syndrome even when they exceed established thresholds for the diagnosis. BNP levels vary with age, sex, body habitus, renal function, and abruptness of symptom onset. Elevated BNP levels also have been associated with renal failure (because of reduced clearance), pulmonary embolism, pulmonary hypertension, and chronic hypoxia. BNP measures are not a substitute for a comprehensive assessment for signs and symptoms of heart failure, and a laboratory test by itself cannot be used to determine the diagnosis or management of heart failure. Clinical evaluation and follow-up are essential to assure proper care for patients with heart failure or any other cardiac problem.
The best drug treatment for symptomatic mitral valve prolapse is:
The primary treatment for symptomatic mitral valve prolapse is *β-blockers* such as *propranolol* (Inderal). Quinidine and digoxin were used to treat this problem in the past, especially if sinus bradycardia or cardiac arrest occurred with administration of propranolol. Asymptomatic patients require only routine monitoring, while those with significant mitral regurgitation may require surgery. Some patients with palpitations can be managed with lifestyle changes such as *elimination of caffeine and alcohol*. Orthostatic hypotension can often be managed with volume expansion, such as by *increasing salt intake*.
Which one of the following is an absolute contraindication to electroconvulsive therapy (ECT)?
There are no absolute contraindications to electroconvulsive therapy (ECT), but factors that have been associated with reduced efficacy include a prolonged episode, lack of response to medication, and coexisting psychiatric diagnoses such as a personality disorder. Persons who may be at increased risk for complications include those with unstable cardiac disease such as ischemia or arrhythmias, cerebrovascular disease such as recent cerebral hemorrhage or stroke, or increased intracranial pressure. ECT can be used safely in elderly patients and in persons with cardiac pacemakers or implantable cardioverter-defibrillators. ECT also can be used safely during pregnancy.
A 60-year-old African-American male who has a 15-year history of diabetes mellitus reports a 1-week history of weakness of the lower left leg, giving way of the knee, and discomfort in the anterior thigh. He has no history of recent trauma. A physical examination reveals *decreased sensation to pinprick and light touch* over the left anterior thigh, and *reduced motor strength* on hip flexion and knee extension. The straight leg raising test is normal. The most likely cause of this condition is
These findings are typical of *femoral neuropathy*, a mononeuropathy commonly associated with diabetes mellitus. *Diabetic polyneuropathy* is characterized by symmetric and distal limb sensory and motor deficits. *Meralgia paresthetica*, or lateral femoral cutaneous neuropathy, may be secondary to diabetes mellitus, but is manifested by numbness and paresthesia over the anterolateral thigh with no motor dysfunction. *Spinal stenosis* causes pain in the legs, but is not associated with the neurologic signs seen in this patient, nor with knee problems. *Iliofemoral atherosclerosis*, a relatively common complication of diabetes mellitus, may produce intermittent claudication involving one or both calf muscles but would not produce the motor weakness noted in this patient.
A 3-day-old female developed a rash 1 day ago that has continued to progress and spread. The infant was born at term after an uncomplicated pregnancy and delivery to a healthy mother following excellent prenatal care. The infant was discharged 2 days ago in good health. She does not appear to be irritable or in distress, and she is afebrile and feeding well. On examination, abnormal findings are confined to the skin, including her face, trunk, and proximal extremities, which have macules, papules, and pustules that are all 2-3 mm in diameter. Her palms and soles are spared. A stain of a pustular smear shows numerous eosinophils. Which one of the following is the most likely diagnosis?
This baby has *Erythema toxicum neonatorum*, a common rash in neonates, occurring in up to half of newborns carried to term, usually between day 2-5 after birth; it does not occur outside the neonatal period. typically resolves within first two weeks of life. The cause is thought to be an activation of the immune system. The fluid from erythema toxicum lesions will show many eosinophils. Because the eruption is transient and self-limiting, no treatment is indicated *Staphylococcal pyoderma* is vesicular and the stain of the vesicle content shows polymorphonuclear leukocytes and clusters of gram-positive bacteria. Because the mother is healthy and the infant shows no evidence of being otherwise ill, systemic infections such as *herpes* are unlikely. *Acne neonatorum *consists of closed comedones on the forehead, nose, and cheeks. *Rocky Mountain spotted fever* is a tickborne disease that does not need to be considered in a child who is not at risk.
An 81-year-old male with type 2 diabetes mellitus has a hemoglobin A 1c of 10.9%. He is already on the maximum dosage of glipizide (Glucotrol). His other medical problems include mild renal insufficiency and moderate ischemic cardiomyopathy. Which one of the following would be the most appropriate change in this patient's diabetes regimen?
This geriatric diabetic patient should be treated with *insulin*. *Metformin is contraindicated in patients with renal insufficiency*. *Sitagliptin* should not be added to a sulfonylurea drug initially, the dosage should be lowered in patients with renal insufficiency, and given alone it would probably not result in reasonable diabetic control. *Pioglitazone* can cause fluid retention and therefore would not be a good choice for a patient with cardiomyopathy.
You examine an 11-month-old male who has had several paroxysms of abdominal pain in the last 2 hours. The episodes last 1-2 minutes; the infant screams, turns pale, and doubles up. Afterward, he seems normal. A physical examination is normal except for a possible fullness in the right upper quadrant of the abdomen. The most likely diagnosis is:
This is a classic presentation for *intussusception*, which usually occurs in children under the age of 2 years and is characterized by paroxysms of colicky abdominal pain. A mass is palpable in about two-thirds of patients. *Pyloric stenosis* presents with a palpable mass, but usually develops between 4 and 6 weeks of age. *Choledochal cyst* presents with the classic triad of right upper quadrant pain, jaundice, and a palpable mass. *Meckel's diverticulum* usually presents in this age group with painless lower gastrointestinal bleeding. *Intestinal malrotation* usually presents within the first 4 weeks of life and is characterized by bilious vomiting.
Patients with rheumatoid arthritis should be screened for tuberculosis before starting which one of the following medications?
Tumor necrosis factor inhibitors have been associated with an increased risk of infections, including tuberculosis. This class of agents includes monoclonal antibodies such as *infliximab*, adalimumab, certolizumab pegol, and golimumab. Patients should be screened for tuberculosis and hepatitis B and C before starting these drugs.
You have just diagnosed mild persistent asthma in a 13-year-old African-American female. Along with patient education, your initial medical management should be:
a low-dose inhaled corticosteroid daily, along with a short-acting inhaled β-agonist as needed Inhaled corticosteroids improve asthma control in adults and children more effectively than any other single long-term controller medication, and all patients should also receive a prescription for a short-acting β-agonist (SOR A).
When an interpreter is needed for a patient with limited English proficiency, which one of the following should be AVOIDED when possible?
Using an educated adult family member who is bilingual. Using trained, qualified interpreters for patients with limited English proficiency leads to fewer hospitalizations, less reliance on testing, a higher likelihood of making the correct diagnosis and providing appropriate treatment, and better patient understanding of conditions and therapies. Although the patient may request that a family member interpret, there are many pitfalls in using untrained interpreters: a lack of understanding of medical terminology, concerns about confidentiality, and unconscious editing by the interpreter of what the patient has said. Additionally, the patient may be reluctant to divulge sensitive or potentially embarrassing information to a friend or family member. The other principles listed are important practices when working with interpreters. Pictures and diagrams can help strengthen the patient's understanding of his or her health care.
A 52-year-old patient is concerned about a biopsy result from a recent screening colonoscopy. Which one of the following types of colon polyp is most likely to become malignant?
Villous adenoma. Hamartomatous (or juvenile) polyps and hyperplastic polyps are benign lesions and are not considered to be premalignant. Adenomas, on the other hand, have the potential to become malignant. Sessile adenomas and lesions >1.0 cm have a higher risk for becoming malignant. Of the three types of adenomas (tubular, tubulovillous, and villous), villous adenomas are the most likely to develop into an adenocarcinoma.
A previously healthy 82-year-old male is brought to your office by his daughter after a recent fall while getting up to go to the bathroom in the middle of the night. The patient denies any history of dizziness, chest pain, palpitations, or current injury. He has a history of bilateral dense cataracts. On examination, he is found to have an increased stance width and walks carefully and cautiously with his arms and legs abducted. A timed up-and-go test is performed, wherein the patient is asked to rise from a chair without using his arms, walk 3 meters, turn, return to his chair, and sit down. It takes the patient 25 seconds and he is noted to have an "en bloc" turn. Which one of the following is the most likely cause of this patient's gait and balance disorder?
Visual impairment The "Timed Up and Go" test is a reliable diagnostic tool for gait and balance disorders and is quick to administer. A time of <10 seconds is considered normal, a time of >14 seconds is associated with an increased risk of falls, and a time of >20 seconds usually suggests severe gait impairment. This patient has the cautious gait associated with visual impairment. It is characterized by abducted arms and legs; slow, careful, "walking on ice" movements; a wide-based stance; and "en bloc" turns. Patients with *cerebellar degeneration* have an ataxic gait that is wide-based and staggering. *Frontal lobe degeneration* is associated with gait apraxia that is described as "magnetic," with start and turn hesitation and freezing. *Parkinson's disease* patients have a typical gait that is short-stepped and shuffling, with hips, knees, and spine flexed, and may also exhibit festination and "en bloc" turns. *Motor neuropathy* causes a "steppage" gait resulting from foot drop with excessive flexion of the hips and knees when walking, short strides, a slapping quality, and frequent tripping
A 20-year-old white female presents with painful and frequent urination that has had a gradual onset over the past week. She has never had a urinary tract infection. There is no associated hematuria, flank pain, suprapubic pain, or fever. She says she has not noted any itching or vaginal discharge. A midstream urine specimen taken earlier in the week showed significant pyuria but a culture was reported as no growth. She has taken an antibiotic for 2 days without relief. Her only other medication is an oral contraceptive agent. Which one of the following is the most likely infectious agent?
Women who present with symptoms of acute dysuria, frequency, and pyuria do not always have bacterial cystitis. In fact, up to 30% will show either no growth or insignificant bacterial growth on a midstream urine culture. Most commonly these patients represent cases of *sexually transmitted urethritis* caused by *Chlamydia trachomatis, Neisseria gonorrhoeae, or herpes simplex virus* In this case, the gradual onset, *absence of hematuria*, and week-long duration of symptoms suggest a sexually transmitted disease. A history of a new sexual partner or a finding of mucopurulent cervicitis would confirm the diagnosis. Empiric treatment with a *tetracycline* and a search for other sexually transmitted diseases would then be indicated. Another possible diagnosis is urinary tract infection with Escherichia coli or Staphylococcus species; however, the onset of these infections is usually abrupt and accompanied by other signs, such as suprapubic pain or hematuria. Candida is unlikely because there is no accompanying discharge or itching, and the patient's symptoms predate the use of antibiotics.
Women who use low-dose estrogen oral contraceptives have a 50% lower risk of cancer of the?
Women who use low-dose estrogen oral contraceptives have at least a 50% lower risk of subsequent *epithelial ovarian cancer* than women who have never used them. Epidemiologic data also suggests other potential long-term benefits of oral contraceptives, including a reduced risk of *postmenopausal fractures*, as well as reductions in the risk of *endometrial and colorectal cancers*. Oral contraceptives do *not* reduce the risk of carcinoma of the breast, cervix, lung, or head and neck.
You see a 6-year-old male for the third time in 3 months with a persistently painful hand condition. He has been treated with oral amoxicillin, followed by oral trimethoprim/ sulfamethoxazole (Bactrim, Septra), with no improvement. A physical examination reveals retraction of the proximal nail fold, absence of the cuticle, and erythema and tenderness around the nail fold area. The thumb and second and third fingers are affected on both hands. The patient is otherwise healthy. First-line treatment for this condition includes:
a *topical corticosteroid cream*. This patient has symptoms and signs consistent with *chronic paronychia*. This condition is often associated with chronic immersion in water, contact with soaps or detergents, use of certain systemic drugs (antiretrovirals, retinoids) and, as is most likely in a 6-year-old child, finger sucking. Findings on examination are similar to those of acute paronychia, with tenderness, erythema, swelling, and retraction of the proximal nail fold. Often the adjacent cuticle is absent. Chronic paronychia has usually been persistent for at least 6 weeks by the time of diagnosis. In addition to medication, basic treatment principles for the condition include avoidance of contact irritants, avoiding immersion of the hands in water, and use of an emollient. Topical corticosteroids have higher efficacy for treating chronic paronychia compared to oral antifungals (SOR B), particularly given the young age of the patient. A topical antifungal can also be tried in conjunction with the corticosteroid.
A 50-year-old female presents with right eye pain. On examination, you find no redness, but when you test her extraocular muscles she reports marked pain with eye movement. This finding suggests that her eye pain is caused by:
an orbital problem Pain with eye movement suggests an orbital condition. Orbital inflammation, infection, or tumor invasion can lead to such eye pain. Other findings suggestive of an orbital cause of eye pain include diplopia or proptosis. If an orbital lesion is suspected, imaging studies should be performed.
A 30-year-old female asks you whether she should have a colonoscopy, as her father was diagnosed with colon cancer at the age of 58. There are no other family members with a history of colon polyps or cancer. You recommend that she have her first screening colonoscopy:
at age 40 and then every 5 years if normal Patients who have one 1st degree relative diagnosed with colorectal cancer or adenomatous polyps before age 60, or at least two 2nd degree relatives with colorectal cancer, are in the highest risk group. They should start colon cancer screening at *age 40, or 10 years before the earliest age* at which an affected relative was diagnosed (whichever comes first) and be rescreened *every 5 years*. Colonoscopy is the preferred screening method for this highest-risk group, as high-risk patients are more likely to have right-sided colon lesions that would not be detected with sigmoidoscopy.
A 75-year-old African-American male with no previous history of cardiac problems complains of shortness of breath and a feeling of general weakness. His symptoms have developed over the past 24 hours. On physical examination you find a regular pulse with a rate of 160 beats/min. You note rales to the base of the scapula bilaterally, moderate jugular venous distention, and hepatojugular reflux. His blood pressure is 90/55 mm Hg; when he sits up he becomes weak and diaphoretic and complains of precordial pressure. An EKG reveals atrial flutter with 2:1 block. Management at this time should include
electrical cardioversion is treatment of choice for symptomatic atrial flutter Atrial flutter is not ordinarily a serious arrhythmia, but this patient has heart failure manifested by rales, jugular venous distention, hepatojugular reflux, hypotension, and angina. Electrical cardioversion should be performed immediately. This is generally a very easy rhythm to convert. Digoxin and verapamil are appropriate in hemodynamically stable patients. A pacemaker for rapid atrial pacing may be beneficial if digitalis intoxication is the cause of atrial flutter, but this is unlikely in a patient with no previous history of cardiac problems. Amiodarone is not indicated in this clinical situation.
A 42-year-old male with well-controlled type 2 diabetes mellitus presents with a 24-hour history of influenza-like symptoms, including the sudden onset of headache, fever, myalgias, sore throat, and cough. It is December, and there have been a few documented cases of influenza recently in the community. The CDC recommends initiating treatment in this situation:
on the basis of clinical symptoms alone The effectiveness of treatment for influenza is dependent on how early in the course of the illness it is given. Antiviral treatment is recommended as soon as possible for patients with confirmed or suspected influenza who have severe, complicated, or progressive illness or who require hospitalization. Antiviral treatment is recommended as soon as possible for outpatients with confirmed or suspected influenza who are at higher risk for influenza complications based on their age or underlying medical conditions. Clinical judgment should be an important component of outpatient treatment decisions. Antiviral treatment also may be considered on the basis of clinical judgment for any outpatient with confirmed or suspected influenza who does not have known risk factors for severe illness, if treatment can be initiated within 48 hours of illness onset. Many rapid influenza tests produce false-negative results, and more accurate assays can take more than 24 hours. Thus, treatment of patients with a clinical picture suggesting influenza is recommended, even if a rapid test is negative. Delaying treatment until further test results are available is not recommended.
A 42-year-old male with a history of intravenous drug use asks to be tested for hepatitis C. The hepatitis C virus (HCV) antibody enzyme immunoassay and recombinant immunoblot assay are both reported as positive. The quantitative HCV RNA polymerase chain reaction test is negative. These test results are most consistent with:
past infection with HCV that is now resolved. There are 3 tests we use for HCV; an initial screen, a confirmatory test and a quantitative test to establish viral load, i.e. active vs past infection The most widely used initial assay for detecting HCV antibody is the *enzyme immunoassay*. A positive enzyme immunoassay should be followed by a confirmatory test such as the *recombinant immunoblot assay*. If negative, it indicates a false-positive antibody test. If positive, the *quantitative HCV RNA PCR* is used to measure the amount of virus in the blood to distinguish active from resolved HCV infection. In this case, the results of the test indicate that the patient had a past infection with HCV that is now resolved.
A painful thrombosed external hemorrhoid diagnosed within the first 24 hours after occurrence is ideally treated by:
thrombectomy under local anesthesia A thrombosed external hemorrhoid is manifested by the sudden development of a painful, tender, perirectal lump. Because there is somatic innervation, the pain is intense, and increases with edema. Treatment involves excision of the acutely thrombosed tissue under local anesthesia, mild pain medication, and sitz baths. It is inappropriate to use procedures that would increase the pain, such as banding or cryotherapy. Total hemorrhoidectomy is inappropriate and unnecessary.
When prescribing an inhaled corticosteroid for control of asthma, the risk of oral candidiasis can be decreased by:
using a valved holding chamber Pharyngeal and laryngeal side effects of inhaled corticosteroids include sore throat, coughing on inhalation of the medication, a weak or hoarse voice, and oral candidiasis. Rinsing the mouth after each administration of the medication and using a valved holding chamber when it is delivered with a metered-dose inhaler can minimize the risk of oral candidiasis.